Demonstre que sendo m inteiro
e positivo a parte inteira de (2+3^1/2)^m e sempre um número ímpar.
Alguém poderia me ajudar
neste problema, agradeço.
Sabendo que x, y, z, t pertencentes
ao naturais. Calcule-os.
31(xyzt+xy+xt+zt+1)=40(yzt+y+t)
Como regra geral, pode confiar no Mathlinks. Estyes
caras tem uma boa experiencia no assunto.
Alias, alguem sabe de algum local na Net onde estejam
os problemas do Torneio das Cidades, disponivel para
download? Esses sao os melhores para quem curte
problemas nem um pouco triviais...
--- Carlos Go
Márcio, se o seu objetivo são problemas para teinamento básico de olimpiadas
pode comprar eu os comprei e não me arrependi, são muito bons,
excelentesproblemas e muito bem organizados
Cgomes
- Original Message -
From: "Marcio" <[EMAIL PROTECTED]>
To:
Sent: Thursday, August 25, 2005 1
Claro..
Seja g = f' entao.
Vou supor que g seja continua em um intervalo I.
Sejam x,y \in I e suponha que x < y. Para n = 0, 1,2... tome
T[n] = { i/(2^n) : i = 0,1,...,2^n}. Vamos mostrar por inducao que para
n = 0 ,1, e para s /in T[n],
g((1-s)x +sy) <= (1-s)g(x) + sg(y).
Se n = 0, entao
Boa tarde, pessoal.
Estive olhando a página www.artofproblemsolving.com e estou querendo
comprar os dois volumes do livro "The Art of Problem Solving", de Sandor
Lehoczky e Richard Rusczyk.
Alguém os recomenda?
[]s,
Márcio.
--
Using Opera's revolutionary e-mail client: http://www.opera.c
A prova que eu achei para esta proposicao, baseada no que li sobre teoria de
medidas, baseia-se nos seguintes fatos:
A condicao (1) g(x+y)/2) <= (g(x) + g(y))/2 para todos reais x e y aliada aa
continuidade de g em R garante que g seja convexa em R. Entretanto, (1)
apenas nao garante continuidade
De fato dah . A condicao f'((x+y)/2) <= (f'(x) + f'(y))/2 sozinha nao
garante garante continuidade, mas esta condicao, aliada ao fato de que f' eh
uma derivada , garante continuidade.
Vc poderia apresentar a prova que vc conhece?
Artur
-Mensagem original-
De: [EMAIL PROTECTED] [mailto:[EM
Eh verdade, mas acho que a propriedade do valor intermediario nao eh
suficiente para garantir a convexidade da derivada.
Artur
-Mensagem original-
De: [EMAIL PROTECTED] [mailto:[EMAIL PROTECTED]
nome de Bernardo Freitas Paulo da Costa
Enviada em: quinta-feira, 25 de agosto de 2005 13:18
Pa
Mais uma vez , eu agradeço a ajuda e agora acho que posso dormir em paz, mesmo depois do árduo trabalho que terei para resolver esta questão.
Valeu"Nicolau C. Saldanha" <[EMAIL PROTECTED]> escreveu:
On Thu, Aug 25, 2005 at 01:19:04AM -0300, Jefferson Franca wrote:> Nem vou perguntar de onde tirou e
Veja que a derivada, mesmo que fosse descontínua, ainda assim
satisfaria a propriedade do valor intermediário. Eu acho que n~ao deve
ser muito difícil concluir a partir disso.
On 8/25/05, Fabio Niski <[EMAIL PROTECTED]> wrote:
> Artur Costa Steiner wrote:
> > Eu achei este problema, um tanto sutil
Artur Costa Steiner wrote:
Eu achei este problema, um tanto sutil, interessante:
Mostre que, se f:R-->R for diferenciavel e sua derivada f' satisfizer a
f'((x+y)/2) <= (f'(x) + f'(y))/2 para todos reais x e y, entao f' eh convexa
em R.
Artur
Antes te pergunto: Será que dá pra afirmar que f'
Ola Pessoal
desta lista ... OBM-L,
A TEORIA DE GALOIS e uma teoria que permite responder com precisao quanto um
determinado numero e ou nao construtivel. E uma teoria simples, mas exige
algum preparo previo, sobretudo em Algebra. O chamado "Teorema de Galois" e
o nucleo da teoria e pressupondo
C(x, 3) = 120
x! / (3! * (x - 3)!) = 120 = 5! ==>x * (x-1) * (x-2) * (x-3)! / (3!
* (x-3)!) = 5! ==> x*(x-1)*(x-2) = 5! * 3! = 720 = 6! = 2^4 * 3^2 *
5 = 5*2 * 3^2 * 2^3 = 10*9*8 ==> x = 10
Logo, o polígono era um decágono.
Abraço
Bruno
On 8/25/05, matduvidas48 <[EMAIL PROTECTED]> wrote:
Algué
Eu achei este problema, um tanto sutil, interessante:
Mostre que, se f:R-->R for diferenciavel e sua derivada f' satisfizer a
f'((x+y)/2) <= (f'(x) + f'(y))/2 para todos reais x e y, entao f' eh convexa
em R.
Artur
=
Instruç
Vou dar um espaço para quem quiser pensar, está depois da mensagem
original do Paulo.
On 8/25/05, Paulo Santa Rita <[EMAIL PROTECTED]> wrote:
> Ola Pessoal
> desta lista ... OBM-L,
>
> Vimos que no caso do primo p=3 para quaisquer R1,R2,R3 pertencentes a {1,2}
> temos sempre que que R1^2 + R2^2 +
Ola Pessoal
desta lista ... OBM-L,
Vimos que no caso do primo p=3 para quaisquer R1,R2,R3 pertencentes a {1,2}
temos sempre que que R1^2 + R2^2 + R3^2 e multiplo de tres, vale dizer, se
tomarmos 3 naturais nao divisiveis por 3, a soma dos seus quadrados e
divisivel por 3.
Veja que no caso p=
Ola Bernardo,
E eu nao disse isso... Apenas o exemplo que usei eraam todos "==1". Mas e
obvio que o raciocinio e o mesmo, tanto no caso "==1", "==2" e suas
combinacoes.
Um Abraco
Paulo Santa Rita
5,1050,250805
From: Bernardo Freitas Paulo da Costa <[EMAIL PROTECTED]>
Reply-To: obm-l@mat.pu
Bem, diga-se de passagem isto e equivalente a testar
para as trincas de naturais abaixo:
(1,1,1)
(1,1,2)
(1,2,2)
(2,2,2)
E e porrada!
--- Paulo Santa Rita <[EMAIL PROTECTED]>
escreveu:
> Ola Renato e demais
> colegas desta lista ... OBM-L,
>
> ( NAO USAREI ACENTOS )
>
> Voce percebeu bem. Co
Ola Carissimo Prof Nicolau e
demais colegas desta lista ... OBM-L,
Em verdade, nao e so o poligono regular de 17 lados que e "estranhamente"
construtivel. Um poligono de N lados e construtivel ( com regua e compasso )
se fi(N) for uma potencia de dois, vale dizer, se N=(2^k)*p1*p2*..*ps, onde
Simples! Teste para as possiveis ternas de naturais
nao multiplas de 3.
Melhor ainda: se n nao e multiplo de 3 entao (n^2-1) e
multiplo de 3.
Uma demo:
(n+1)n(n-1) e multiplo de 3, pois sao 3 naturais
consecutivos, e nao tem como nenhum deles nao ser um
multiplo de 3
Pela hipotese nos sabemos que
Cuidado: nada obriga a == b == c (usando a sua notaç~ao para
congruência) módulo 3. Eu acho que (para n~ao ter que fazer as 2^3
contas) dá pra fazer assim:
Se x == 1 (mod 3), x^2 == 1 (mod 3).
Se x == 2 == -1 (mod 3), x^2 == 1 (mod 3).
Daí, se x !== 0 (mod 3) , ou seja, x n~ao é múltiplo de 3, temo
Bem, se eu nao me engano foi mais ou menos esta a
ideia de Gauss.
--- Jefferson Franca <[EMAIL PROTECTED]>
escreveu:
> Nem vou perguntar de onde tirou essa idéia, mas
> valeu pela solução.
>
> "Nicolau C. Saldanha" <[EMAIL PROTECTED]>
> escreveu:On Mon, Aug 22, 2005 at 01:26:34PM -0300,
> Jeffe
Aguinaldo,
Infelizmente este mês estou meio desprevenido. No mes que vem poderia dar
alguma ajuda.
Abraço,
Anderson
At 15:22 22/8/2005, aguinaldo goncalves jr wrote:
Pessoal,
Será que alguém poderia me ajudar Tenho interesse em adquirir o livro
de álgebra do Morgado (segundo grau) volume
On Thu, Aug 25, 2005 at 01:19:04AM -0300, Jefferson Franca wrote:
> Nem vou perguntar de onde tirou essa idéia, mas valeu pela solução.
Em um curso de álgebra que cubra teoria de Galois este tipo de coisa
é explicada com mais contexto. Eu dei um esboço rápido e elementar.
Aliás, todos estes "prov
Ola Renato e demais
colegas desta lista ... OBM-L,
( NAO USAREI ACENTOS )
Voce percebeu bem. Como ser suficientemente geral ? Da forma como voce fez
...
OBS : "==" significa "e congruente a"
a = 3N+1, N natural => a == 1 (mod 3 ) => a^2 == 1^2 (mod 3)
b = 3N+1, N natural => b ==
Bom dia!
Recentemente me deparei com o seguinte problema,
bastante curioso:
"Mostre que se a, b e c são números naturais não
divisíveis por 3, então a^2 + b^2 + c^2 é divisível por 3."
Pensei em equacionar um natural não divisivel por
tres como 3n+1 ou 3n+2, sendo n natural também.
Ora, m
27 matches
Mail list logo